A system of experimental homework in physics using children's toys. Experimental work to develop the ability of vocational school students to solve problems in physics


The importance and types of independent experiment of students in physics. When teaching physics in high school, experimental skills are developed by performing independent laboratory work.

Teaching physics cannot be presented only in the form of theoretical classes, even if students are shown physical demonstration experiments in class. To all types of sensory perception, it is imperative to add “work with your hands” in classes. This is achieved when students perform a laboratory physical experiment, when they themselves assemble installations, measure physical quantities, and perform experiments. Laboratory classes arouse very great interest among students, which is quite natural, since in this case the student learns about the world around him on the basis of his own experience and his own feelings.

The importance of laboratory classes in physics lies in the fact that students develop ideas about the role and place of experiment in knowledge. When performing experiments, students develop experimental skills, which include both intellectual and practical skills. The first group includes the skills to: determine the purpose of the experiment, put forward hypotheses, select instruments, plan an experiment, calculate errors, analyze the results, draw up a report on the work done. The second group includes the skills to assemble an experimental setup, observe, measure, and experiment.

In addition, the significance of the laboratory experiment lies in the fact that when performing it, students develop such important personal qualities as accuracy in working with instruments; maintaining cleanliness and order in the workplace, in the notes made during the experiment, organization, persistence in obtaining results. They develop a certain culture of mental and physical labor.

In the practice of teaching physics at school, three types of laboratory classes have developed:

Frontal laboratory work in physics;

Physical workshop;

Home experimental work in physics.

Front laboratory work- this is the kind practical work when all students in a class simultaneously perform the same type of experiment using the same equipment. Frontal laboratory work is most often carried out by a group of students consisting of two people; sometimes it is possible to organize individual work. Accordingly, the office should have 15-20 sets of instruments for frontal laboratory work. The total number of such devices will be about a thousand pieces. The names of frontal laboratory work are given in the curriculum. There are quite a lot of them, they are provided for almost every topic of the physics course. Before carrying out the work, the teacher identifies the students’ readiness to consciously carry out the work, determines its purpose with them, discusses the progress of the work, the rules for working with instruments, and methods for calculating measurement errors. Front-end laboratory work is not very complex in content, is closely related chronologically to the material being studied and, as a rule, is designed for one lesson. Descriptions of laboratory work can be found in school textbooks in physics.

Physics workshop carried out with the aim of repeating, deepening, expanding and generalizing the knowledge gained from different topics physics course; development and improvement of students' experimental skills through the use of more complex equipment, more complex experiments; formation of their independence in solving problems related to the experiment. Physics workshop is not related in time to the material being studied; it is usually held at the end school year, sometimes at the end of the first and second half of the year and includes a series of experiments on a particular topic. Students perform physical practical work in a group of 2-4 people using various equipment; During the next classes there is a change of work, which is done according to a specially designed schedule. When drawing up a schedule, take into account the number of students in the class, the number of workshops, and the availability of equipment. Two teaching hours are allocated for each physics workshop, which requires the introduction of double physics lessons into the schedule. This presents difficulties. For this reason and due to the lack necessary equipment practice one-hour physical practicum work. It should be noted that two-hour work is preferable, since the work of the workshop is more complex than frontal laboratory work, they are performed on more complex equipment, and the share of independent participation of students is much greater than in the case of frontal laboratory work. Physical workshops are provided mainly by the programs of grades 9-11. In each class, approximately 10 hours of instructional time are allocated for the workshop. For each work, the teacher must draw up instructions, which should contain: title, purpose, list of instruments and equipment, brief theory, description of devices unknown to students, plan for completing the work. After completing the work, students must submit a report, which must contain: the title of the work, the purpose of the work, a list of instruments, a diagram or drawing of the installation, a plan for performing the work, a table of results, formulas by which the values ​​of quantities were calculated, calculations of measurement errors, conclusions. When assessing the work of students in a workshop, one should take into account their preparation for work, a report on the work, the level of development of skills, understanding of theoretical material, and the experimental research methods used.

Home experimental work. Home laboratory work is the simplest independent experiment that is carried out by students at home, outside of school, without direct supervision by the teacher over the progress of the work.

The main objectives of experimental work of this type are:

Formation of the ability to observe physical phenomena in nature and in everyday life;

Formation of the ability to carry out measurements using measuring instruments used in everyday life;

Formation of interest in experiments and in the study of physics;

Formation of independence and activity.

Home laboratory work can be classified depending on the equipment used to perform it:

Works that use household items and available materials (measuring cup, tape measure, household scales, etc.);

Works in which homemade instruments are used (lever scales, electroscope, etc.);

Work performed on devices produced by industry.

Classification taken from.

In his book S.F. Pokrovsky showed that home experiments and observations in physics conducted by the students themselves: 1) enable our school to expand the area of ​​connection between theory and practice; 2) develop students’ interest in physics and technology; 3) awaken creative thought and develop the ability to invent; 4) teach students to be independent research work; 5) develop valuable qualities in them: observation, attention, perseverance and accuracy; 6) supplement classroom laboratory work with material that cannot be completed in class (a series of long-term observations, observation natural phenomena etc.), and 7) accustom students to conscious, purposeful work.

Home experiments and observations in physics have their own characteristic features, being an extremely useful addition to classroom and school practical work in general.

It has long been recommended that students have a home laboratory. it included, first of all, rulers, a beaker, a funnel, scales, weights, a dynamometer, a tribometer, a magnet, a watch with a second hand, iron filings, tubes, wires, a battery, and a light bulb. However, despite the fact that the set includes very simple devices, this proposal has not gained popularity.

To organize home experimental work for students, you can use the so-called mini-laboratory proposed by teacher-methodologist E.S. Obedkov, which includes many household items (penicillin bottles, rubber bands, pipettes, rulers, etc.) that is available to almost every schoolchild. E.S. Obyedkov developed a very large number of interesting and useful experiments with this equipment.

It also became possible to use a computer to conduct a model experiment at home. It is clear that the corresponding tasks can only be offered to those students who have a computer and software and pedagogical tools at home.

In order for students to want to learn, the learning process must be interesting for them. What is interesting to students? To get an answer to this question, let us turn to excerpts from the article by I.V. Litovko, MOS(P)Sh No. 1, Svobodny “Home experimental tasks as an element of student creativity”, published on the Internet. This is what I.V. writes. Litovko:

“One of the most important tasks of the school is to teach students to learn, to strengthen their ability for self-development in the educational process, for which it is necessary to form in schoolchildren the corresponding stable desires, interests, and skills. An important role in this is played by experimental tasks in physics, which in their content represent short-term observations, measurements and experiments that are closely related to the topic of the lesson. The more observations of physical phenomena and experiments a student makes, the better he will understand the material being studied.

To study students’ motivation, they were asked the following questions and the results were obtained:

What do you like about studying physics? ?

a) problem solving -19%;

b) demonstration of experiments -21%;

EXPERIMENTAL

TASKS

DURING TRAINING

PHYSICISTS

Sosina Natalia Nikolaevna

Physics teacher

MBOU "Central Educational Center No. 22 - Lyceum of Arts"

Experimental problems play a big role in students' learning in physics. They develop thinking and cognitive activity, contribute to a deeper understanding of the essence of phenomena, and develop the ability to build a hypothesis and test it in practice. The main importance of solving experimental problems lies in the formation and development with their help of observation, measuring skills, and ability to handle instruments. Experimental tasks help to increase student activity in lessons and develop logical thinking, teach to analyze phenomena.

Experimental problems include those that cannot be solved without experiments or measurements. These problems can be divided into several types according to the role of experiment in the solution:

    Problems in which it is impossible to obtain an answer to the question without experiment;

    An experiment is used to create a problem situation;

    An experiment is used to illustrate a phenomenon about which we're talking about in the task;

    An experiment is used to verify the correctness of the solution.

You can solve experimental problems both in class and at home.

Let's look at some experimental problems that can be used in the classroom.

SOME CHALLENGING EXPERIMENTAL TASKS

    Explain the observed phenomenon

- If you heat the air in a jar and place a slightly inflated balloon with water on top of the neck of the jar, it will be sucked into the jar. Why?

(The air in the jar cools, its density increases, and its volume

decreases - the ball is drawn into the jar)

- If you pour water on a slightly inflated balloon hot water, then it will increase in size. Why?

(The air heats up, the speed of the molecules increases and they hit the walls of the ball more often. The air pressure increases. The shell is elastic, the pressure force stretches the shell and the ball increases in size)

- A rubber ball placed in a plastic bottle cannot be inflated. Why? What needs to be done to be able to inflate the balloon?

(The ball isolates the air atmosphere in the bottle. As the volume of the ball increases, the air in the bottle is compressed, the pressure increases and prevents the ball from inflating. If a hole is made in the bottle, the air pressure in the bottle will be equal to atmospheric pressure and the ball can be inflated).

- Is it possible to boil water in a matchbox?

    Calculation problems

- How to determine the loss of mechanical energy during one complete oscillation of the load?

(The energy loss is equal to the difference in the potential energy of the load in the initial and final positions after one period).

(To do this, you need to know the mass of the match and its burning time).

    Experimental tasks that encourage information seeking

to answer the question

- Bring a strong magnet to the head of the match, it is almost not attracted. Burn the sulfur head of the match and bring it to the magnet again. Why is the head of the match now attracted to the magnet?

Find information about the composition of a match head.

HOME EXPERIMENTAL TASKS

Experimental problems at home are of great interest to students. By making observations of any physical phenomenon, or performing an experiment at home that needs to be explained when completing these tasks, students learn to think independently and develop their practical skills. Performing experimental tasks plays a particularly important role in adolescence, since during this period the character is rebuilt educational activities schoolboy. A teenager is no longer always satisfied that the answer to his question is in a textbook. He has a need to obtain this answer from life experience, observations of the surrounding reality, from the results of his own experiments. Students complete home experiments and observations, laboratory work, and experimental tasks more willingly and with greater interest than other types of homework. The tasks become more meaningful, deeper, and interest in physics and technology increases. The ability to observe, experiment, explore and construct become integral part in preparing students for further creative work in various fields of production.

Requirements for home experiments

First of all, this is, of course, safety. Since the experiment is carried out by the student at home independently without the direct supervision of the teacher, the experiment should not contain any chemicals or objects that pose a threat to the health of the child and his home environment. The experiment should not require any significant material costs from the student; when conducting the experiment, objects and substances that are found in almost every home should be used: dishes, jars, bottles, water, salt, and so on. An experiment performed at home by schoolchildren should be simple in execution and equipment, but, at the same time, be valuable in the study and understanding of physics in childhood, and be interesting in content. Since the teacher does not have the opportunity to directly control the experiment performed by students at home, the results of the experiment must be formalized accordingly (approximately as is done when performing front-line laboratory work). The results of the experiment carried out by students at home should be discussed and analyzed in class. Students' work should not be a blind imitation of established patterns; they should contain the broadest manifestation own initiative, creativity, searching for something new. Based on the above, we can formulate the requirements for home experimental assignments:

– safety during carrying out;
– minimal material costs;
– ease of implementation;
– have value in the study and understanding of physics;
– ease of subsequent control by the teacher;
– the presence of creative coloring.

SOME EXPERIMENTAL TASKS AT HOME

- Determine the density of a chocolate bar, a bar of soap, a juice bag;

- Take a saucer and lower it edgewise into a pan of water. The saucer is sinking. Now lower the saucer onto the water with its bottom, it floats. Why? Determine the buoyant force acting on the floating saucer.

- Make a hole in the bottom of the plastic bottle with an awl, quickly fill it with water and close the lid tightly. Why did the water stop pouring out?

- How to determine the muzzle velocity of a toy gun bullet using only a tape measure.

- The lamp cylinder says 60 W, 220 V. Determine the resistance of the spiral. Calculate the length of the lamp spiral if it is known that it is made of tungsten wire with a diameter of 0.08 mm.

- Write down the power of the electric kettle according to the passport. Determine the amount of heat released in 15 minutes and the cost of energy consumed during this time.

To organize and conduct a lesson with problematic experimental tasks, the teacher has an opportunity to great opportunity show your creative abilities, choose tasks at your discretion, designed for a particular class, depending on the level of preparation of the students. Currently, there is a large amount of methodological literature that a teacher can rely on when preparing for lessons.

You can use books such as

L. A. Gorev. Entertaining experiments in physics in grades 6-7 of secondary school - M.: “Prosveshcheniye”, 1985

V. N. Lange. Experimental physical tasks for ingenuity: Training manual. - M.: Nauka. Main editorial office of physical and mathematical literature, 1985

L. A. Gorlova. Non-traditional lessons, extracurricular activities– M.: “Vako”, 2006

V. F. Shilov. Home experimental assignments in physics. 7 – 9 grades. – M.: “School Press”, 2003

Some experimental problems are given in the appendices.

ANNEX 1

(from the website of physics teacher V.I. Elkin)

Experimental tasks

1 . Determine how many drops of water are contained in a glass if you have a pipette, scales, a weight, a glass of water, a vessel.

Solution. Pour, say, 100 drops into an empty vessel and determine their mass. How many times the mass of water in a glass is greater than the mass of 100 drops is the number of drops.

2 . Determine the area of ​​a homogeneous piece of irregularly shaped cardboard if you have scissors, a ruler, scales, and weights.

Solution. Weigh the record. Cut out a regular shape from it (for example, a square), the area of ​​which is easy to measure. Find the mass ratio - it is equal to the area ratio.

3 . Determine the mass of a homogeneous cardboard of the correct shape (for example, a large poster), if you have scissors, a ruler, scales, and weights.

Solution. There is no need to weigh the entire poster. Determine its area, and then cut out a regular shape from the edge (for example, a rectangle) and measure its area. Find the area ratio - it is equal to the mass ratio.

4 . Determine the radius of the metal ball without using a caliper.

Solution. Determine the volume of the ball using a beaker, and from the formula V = (4/3) R 3 determine its radius.

Solution. Wind tightly around a pencil, for example, 10 turns of thread and measure the length of the winding. Divide by 10 to find the diameter of the thread. Using a ruler, determine the length of the coil, divide it by the diameter of one thread and get the number of turns in one layer. Having measured the outer and inner diameters of the coil, find their difference, divide by the diameter of the thread - you will find out the number of layers. Calculate the length of one turn in the middle part of the spool and calculate the length of the thread.

Equipment. Beaker, test tube, glass of cereal, glass of water, ruler.

Solution. Consider the grains to be approximately equal and spherical. Using the row method, calculate the diameter of the grain and then its volume. Pour water into the test tube with cereal so that the water fills the gaps between the grains. Using a beaker, calculate the total volume of the cereal. Dividing the total volume of the cereal by the volume of one grain, count the number of grains.

7 . In front of you is a piece of wire, a measuring ruler, wire cutters and a scale with weights. How to cut two pieces of wire at once (with an accuracy of 1 mm) in order to obtain homemade weights weighing 2 and 5 g?

Solution. Measure the length and weight of all the wire. Calculate the length of the wire per gram of its mass.

8 . Determine the thickness of your hair.

Solution. Wind coil to coil of hair onto the needle and measure the length of the row. Knowing the number of turns, calculate the diameter of the hair.

9 . There is a legend about the founding of the city of Carthage. Dido, the daughter of the Tyrian king, having lost her husband who was killed by her brother, fled to Africa. There she bought from the Numidian king as much land “as an oxhide occupies.” When the deal was completed, Dido cut the oxhide into thin strips and, thanks to this trick, covered a plot of land sufficient to build a fortress. So, it seems, the fortress of Carthage arose, and subsequently the city was built. Try to determine approximately how much area the fortress could occupy, if we assume that the size of the cowhide is 4 m2, and the width of the straps into which Dido cut it is 1 mm.

Answer. 1 km 2.

10 . Find out if the aluminum object (such as a ball) has a cavity inside.

Solution. Using a dynamometer, determine the weight of the body in air and water. In air P = mg, and in water P = mg – F, where F = gV is the Archimedes force. Using the reference book, find and calculate the volume of the ball V in air and water.

11 . Calculate the internal radius of a thin glass tube using a balance, a measuring ruler, or a container of water.

Solution. Fill the tube with water. Measure the height of the liquid column, then pour the water out of the tube and determine its mass. Knowing the density of water, determine its volume. From the formula V = SH = R 2 H, calculate the radius.

12 Determine the thickness of the aluminum foil without using a micrometer or caliper.

Solution. Determine the mass of the aluminum sheet by weighing, and the area using a ruler. Using a reference book, find the density of aluminum. Then calculate the volume and from the formula V = Sd - the thickness of the foil d.

13 . Calculate the mass of bricks in the wall of the house.

Solution. Since the bricks are standard, look for bricks in the wall whose length, thickness or width can be measured. Using a reference book, find the density of the brick and calculate the mass.

14 . Make a “pocket” scale to weigh liquid.

Solution. The simplest “scale” is a beaker.

15 . Two students made a task to determine the direction of the wind using a weather vane. On top they placed beautiful flags cut from the same piece of tin - on one weather vane a rectangular shape, on the other a triangular one. Which flag, triangular or rectangular, requires more paint?

Solution. Since the flags are made from the same piece of tin, it is enough to weigh them; the larger one has a larger area.

16 . Cover a piece of paper with a book and jerk it up. Why does a leaf rise behind it?

Answer. A piece of paper raises atmospheric pressure because... at the moment the book is torn off, a vacuum is formed between it and the leaf.

17 . How to pour water from a jar on the table without touching it?

Equipment. A three-liter jar, 2/3 filled with water, a long rubber tube.

Solution. Place one end of a long rubber tube completely filled with water into the jar. Take the second end of the tube into your mouth and suck out the air until the level of liquid in the tube is above the edge of the jar, then remove it from your mouth, and lower the second end of the tube below the water level in the jar - the water will flow by itself. (This technique is often used by drivers when pouring gasoline from a car tank into a canister).

18 . Determine the pressure exerted by a metal block lying tightly on the bottom of a vessel with water.

Solution. The pressure on the bottom of the glass is the sum of the pressure of the liquid column above the block and the pressure exerted on the bottom directly by the block. Using a ruler, determine the height of the liquid column, as well as the area of ​​the edge of the block on which it lies.

19 . Two balls of equal mass are immersed, one in clean water, the other in heavily salt water. The lever to which they are suspended is in balance. Determine which container contains clean water. You cannot taste the water.

Solution. A ball immersed in salt water loses less weight than a ball in clean water. Therefore, its weight will be greater, therefore, it is the ball that hangs on the shorter arm. If you remove the glasses, the ball suspended from the longer arm will be pulled.

20 . What needs to be done to make a piece of plasticine float in water?

Solution. Make a “boat” from plasticine.

21 . A plastic soda bottle was filled 3/4 with water. What needs to be done so that a plasticine ball thrown into a bottle will sink, but will float up if the cork is twisted and the walls of the bottle are compressed?

Solution. You need to make an air cavity inside the ball.

22 . What pressure does a cat (dog) exert on the floor?

Equipment. A piece of checkered paper (from a student's notebook), a saucer with water, household scales.

Solution. Weigh the animal on a home scale. Wet his paws and make him run along a piece of squared paper (from a student's notebook). Determine the paw area and calculate the pressure.

23 . To quickly pour the juice out of the jar, you need to make two holes in the lid. The main thing is that when you start pouring the juice from the jar, they should be one at the top, the other diametrically at the bottom. Why are two holes needed and not one? Explanation. Air enters the top hole. Under the influence of atmospheric pressure, juice flows out from the bottom. If there is only one hole, then the pressure in the jar will periodically change, and the juice will begin to “gurgle.”

24 . A hexagonal pencil with a side width of 5 mm rolls along a sheet of paper. What is the trajectory of its center? Draw it.

Solution. The trajectory is a sinusoid.

25 . A dot was placed on the surface of the round pencil. The pencil was placed on an inclined plane and allowed to roll down while rotating. Draw the trajectory of the point relative to the table surface, magnified 5 times.

Solution. The trajectory is a cycloid.

26 . Hang the metal rod on two tripods so that its movement can be progressive; rotational.

Solution. Hang the rod on two threads so that it is horizontal. If you push it along, it will move while remaining parallel to itself. If you push it across, it will begin to oscillate, i.e. make a rotational movement.

27 . Determine the speed of movement of the end of the second hand wristwatch.

Solution. Measure the length of the second hand - this is the radius of the circle along which it moves. Then calculate the circumference, and calculate the speed

28 . Determine which ball has the most mass. (You cannot pick up the balls.)

Solution. Place the balls in a row and, using a ruler, simultaneously give everyone the same push force. The one that flies the shortest distance is the heaviest.

29 . Determine which of two seemingly identical springs has a greater stiffness coefficient.

Solution. Interlock the springs and stretch them in opposite directions. A spring with a lower stiffness coefficient will stretch more.

30 . You are given two identical rubber balls. How can you prove that one of the balls will bounce higher than the other if they are dropped from the same height? Throwing balls, pushing them against each other, lifting them from the table, rolling them around the table is prohibited.

Solution. You need to press the balls with your hand. Whichever ball is more elastic will bounce higher.

31 . Determine the coefficient of sliding friction of a steel ball on wood.

Solution. Take two identical balls, connect them together with plasticine so that they do not rotate when rolling. Place a wooden ruler in a tripod at such an angle that the balls sliding along it move straight and evenly. In this case = tg, where is the angle of inclination. By measuring the height of the inclined plane and the length of its base, find the tangent of this angle of inclination (sliding friction coefficient).

32 . You have a toy gun and a ruler. Determine the speed of the “bullet” when fired.

Solution. Make a shot vertically upward, note the height of the rise. At the highest point, kinetic energy is equal to potential energy - from this equality find the speed.

33 . A horizontally located rod with a mass of 0.5 kg rests at one end on a support and at the other on a removable table of a demonstration dynamometer. What are the dynamometer readings?

Solution. Total weight rod 5 N. Since the rod rests on two points, the weight of the body is distributed equally across both points of support, therefore, the dynamometer will show 2.5 N.

34 . On the student's desk there is a cart with a load. The student pushes it slightly with his hand, and the cart, after traveling some distance, stops. How to find the initial speed of the cart?

Solution. The kinetic energy of the cart at the initial moment of its movement is equal to the work done by the friction force along the entire path of movement, therefore, m 2 /2 = Fs. To find the speed, you need to know the mass of the cart with the load, the friction force and the distance traveled. Based on this, you need to have scales, a dynamometer, and a ruler.

35 . There is a ball and a cube made of steel on the table. Their masses are the same. You lifted both bodies and pressed them to the ceiling. Will they have the same potential energy?

Solution. No. The center of gravity of the cube is lower than the center of gravity of the ball, therefore, the potential energy of the ball is less.

APPENDIX 2

(from the book by V. N. Lange “Experimental physical tasks for ingenuity” - experimental tasks at home)

1. You were asked to find the density of sugar. How to do this, having only a household beaker, if the experiment needs to be carried out with granulated sugar?

2. Using a 100-gram weight, a triangular file and a graduated ruler, how can you approximately determine the mass of a certain body if it does not differ much from the mass of the weight? What to do if instead of a weight you are given a set of “copper” coins?

3. How to use copper coins find the mass of the ruler?

4. The scale of the scales available in the house is graduated only up to 500 g. How can you use them to weigh a book whose mass is about 1 kg, also having a spool of thread?

5. At your disposal are a bathtub filled with water, a small jar with a wide neck, a few pennies, a pipette, and colored chalk (or a soft pencil). How can you use these - and only these - objects to find the mass of one drop of water?

6. How can you determine the density of a stone using scales, a set of weights and a vessel with water if its volume cannot be measured directly?

7. How can you tell, given a spring (or a strip of rubber), twine and a piece of iron, which of two opaque vessels contains kerosene, and which contains kerosene and water?

8. How can you find the capacity (i.e., internal volume) of a pan using scales and a set of weights?

9. How to divide the contents of a cylindrical glass, filled to the brim with liquid, into two identical parts, having another vessel, but of a different shape and slightly smaller volume?

10. Two comrades were relaxing on the balcony and thinking about how to determine, without opening matchboxes, whose box had fewer matches left. What method can you suggest?

11. How to determine the position of the center of mass of a smooth stick without using any tools?

12. How to measure the diameter of a soccer ball using a rigid (for example, regular wooden) ruler?

13. How to find the diameter of a small ball using a beaker?

14. It is necessary to find out the diameter of a relatively thin wire as accurately as possible, having for this purpose only a school notebook “in a square” and a pencil. What should I do?

15. There is a rectangular vessel partially filled with water, in which a body immersed in water floats. How can you find the mass of this body using one ruler?

16. How to find the density of cork using a steel knitting needle and a beaker of water?

17. How, having only a ruler, can you find the density of the wood from which a stick is made floating in a narrow cylindrical vessel?

18. The glass stopper has a cavity inside. Is it possible to determine the volume of a cavity using scales, a set of weights and a vessel with water without breaking the plug? And if it is possible, then how?

19. There is an iron sheet nailed to the floor, a light wooden stick (rod) and a ruler. Develop a method for determining the coefficient of friction between wood and iron using only the items listed.

20. Being in a room illuminated by an electric lamp, you need to find out which of two converging lenses with the same diameters has greater optical power. No special equipment is provided for this purpose. Indicate a way to solve the problem.

21. There are two lenses with the same diameters: one is converging, the other is diverging. How to determine which of them has greater optical power without resorting to instruments?

22. In a long corridor, devoid of windows, there is an electric lamp. It can be lit and extinguished with a switch installed at the entrance door at the beginning of the corridor. This is inconvenient for those who go outside, since they have to make their way in the dark before going out. However, the one who entered and turned on the lamp at the entrance is also dissatisfied: after passing through the corridor, he leaves the lamp burning in vain. Is it possible to come up with a circuit that allows you to turn the lamp on and off from different ends of the corridor?

23. Imagine that you were asked to use an empty tin can and a stopwatch to measure the height of a house. Would you be able to cope with the task? Tell me how to proceed?

24. How to find the speed of flow of water from a water tap, having a cylindrical jar, a stopwatch and a caliper?

25. Water flows out in a thin stream from a loosely closed water tap. How, using only one ruler, can you determine the flow rate of water, as well as its volumetric flow rate (i.e., the volume of water flowing from the tap per unit time)?

26. It is proposed to determine the acceleration of gravity by observing a stream of water flowing from a loosely closed water tap. How to complete the task, having for this purpose a ruler, a vessel of known volume and a clock?

27. Let's say that you need to fill a large tank of known volume with water using a flexible hose equipped with a cylindrical nozzle. You want to know how long this boring activity will last. Is it possible to calculate it with only a ruler?

28. How can you determine the mass of an object using a weight of known mass, a light cord, two nails, a hammer, a piece of plasticine, mathematical tables and a protractor?

29. How to determine the pressure in a soccer ball using a sensitive scale and ruler?

30. How can you determine the pressure inside a burnt-out light bulb using a cylindrical vessel with iodine and a ruler?

31. Try to solve the previous problem if we are allowed to use a pan filled with water and a scale with a set of weights.

32. Given a narrow glass tube, sealed at one end. The tube contains air separated from the surrounding atmosphere by a column of mercury. There is also a millimeter ruler. Use them to determine atmospheric pressure.

33. How to determine the specific heat of vaporization of water, having a home refrigerator, a saucepan of unknown volume, a clock and an evenly burning gas burner? The specific heat capacity of water is assumed to be known.

34. You need to find out the power consumed from the city network by a TV (or other electrical appliance) using a table lamp, a spool of thread, a piece of iron and an electric meter. How to complete this task?

35. How to find the resistance of an electric iron in operating mode (there is no information about its power) using an electric meter and a radio receiver? Consider separately the cases of radios powered by batteries and the city network.

36. It’s snowing outside the window, but it’s warm in the room. Unfortunately, there is nothing to measure the temperature with - there is no thermometer. But there is a battery of galvanic cells, a very accurate voltmeter and ammeter, as much copper wire as you like, and a physical reference book. Is it possible to use them to find the air temperature in the room?

37. How to solve the previous problem if there is no physical reference book, but in addition to the listed items, you are allowed to use an electric stove and a pot of water?

38. The pole designations of the horseshoe magnet at our disposal have been erased. Of course, there are many ways to find out which one is southern and which is northern. But you are asked to complete this task using the TV! What should you do?

39. How to determine the pole signs of an unmarked battery using a coil of insulated wire, an iron rod and a TV.

40. How can you tell if a steel rod is magnetized, given a piece of copper wire and a spool of thread?

41. The daughter turned to her father, who was recording the electric meter readings by lamplight, with a request to let her go for a walk. Giving permission, the father asked his daughter to return in exactly an hour. How can a father control the duration of a walk without using a watch?

42. Problem 22 is published quite often in various collections and is therefore well known. Here is a task of the same nature, but somewhat more complex. Design a circuit that allows you to turn a light bulb or some other electrically powered device on and off from any number of different points.

43. If you place a wooden cube on a cloth-covered disk of a radiogram player close to the axis of rotation, the cube will rotate along with the disk. If the distance to the axis of rotation is large, the cube, as a rule, is thrown off the disk. How to determine the coefficient of friction of wood on cloth using just a ruler?

44. Develop a method for determining the volume of a room using a sufficiently long and thin thread, a clock and a weight.

45. When teaching music, ballet art, training athletes and for some other purposes, a metronome is often used - a device that produces periodic abrupt clicks. The duration of the interval between two beats (clicks) of the metronome is regulated by moving the weight on a special swinging scale. How to graduate the metronome scale in seconds using a thread, a steel ball and a tape measure if this is not done at the factory?

46. ​​The weight of a metronome with a non-graduated scale (see the previous problem) must be set in such a position that the time interval between two beats is equal to one second. For this purpose, you are allowed to use a long ladder, a stone and a tape measure. How should you use this set of items to complete the task?

47. There is a wooden cuboid, in which one edge is significantly larger than the other two. How to use a ruler alone to determine the coefficient of friction of a block on the floor surface in a room?

48. Modern coffee grinders are driven by a low-power electric motor. How to determine the direction of rotation of the rotor of its motors without disassembling the coffee grinder

49. Two hollow balls having the same mass and volume are painted with the same paint, which is not advisable to scratch. One ball is made of aluminum and the other is made of copper. What is the easiest way to tell which ball is aluminum and which is copper?

50. How to determine the mass of a certain body using a uniform rod with divisions and a piece of not very thick copper wire? It is also allowed to use a physical reference book.

51. How to estimate the radius of a concave spherical mirror (or radius of curvature concave lens) using a stopwatch and a steel ball of known radius?

52. Two identical spherical glass flasks are filled with different liquids. How to determine in which liquid the speed of light is greater, having only an electric light bulb and a sheet of paper for this purpose?

53. Dyed cellophane film can be used as a simple monochromator - a device that isolates a rather narrow range of light waves from a continuous spectrum. How can you determine the average wavelength from this interval using a table lamp, a record player with a record (preferably a long-playing one), a ruler and a sheet of cardboard with a small hole? It’s good if a friend with a pencil participates in your experiment.

The effectiveness of using experimental tasks in lessons is largely determined by their manufacturability, unpretentious equipment, and the breadth of the phenomena under consideration. Based on the simplest equipment and even everyday objects, the experimental task brings physics closer to us, transforming it in the students’ minds from an abstract system of knowledge in science that studies “the world around us.”

Mechanics

Task 1. Friction coefficient

Exercise. Measure the sliding friction coefficient of a wooden block on the surface of a board (ruler).

Equipment: block, board, tripod with foot, ruler 30(40) long cm.

Possible solution. We place the block on the board, in accordance with Figure 4. Gradually raising one end of the board, we obtain an inclined plane and achieve uniform sliding of the block. Since the static friction force is much greater than the sliding friction force, it is necessary to push the bead a little at the beginning of sliding. To fix the desired tilt, use a tripod. We measure the height A and the length of the base of the inclined plane b.

Measurements and error analysis:

We repeat the experiment several times. IN in this case this must be done mainly because it is difficult to achieve uniform sliding of the block along the plane. The results are recorded in Table 2.

table 2

Measurement errors

a, cm

Yes, cm

(Yes) 2 ,cm 2

in, cm

Db, cm

(Db) 2 ,cm 2

<a>=12,2

U( a) 2 = 1,81

U( b) 2 = 0,32

In addition to random errors, the total error, of course, also includes the usual reference errors: Yes = Db = 0.5 cm.This amounts to:

Thus we get:

a = 12.2 ± 1.1 cm, d = 8.6%

b = 27.4 ± 0.7 cm, d = 2.6%

Based on the results of the first experiment:

The final result of the friction coefficient measurement is:

m = 0.46 ± 0.05 d = 10.9%

Task 2. Measuring the height of a house

Exercise. Imagine that you were asked to use an empty tin can and a stopwatch to measure the height of a house. Would you be able to cope with the task? Tell us how to act.

Clue. If a can is thrown from the roof of a house, the sound of the can hitting the ground will be clearly audible.

Solution. Standing on the roof of the house, you need to release the can from your hands while simultaneously pressing the stopwatch start button. When you hear the sound of the can hitting the ground, you should stop the stopwatch. Stopwatch readings t are made up of the time the jar falls t 1 and time t 2, during which the sound of its impact on the earth’s surface will reach the observer.

The first time is related to the height of the house h in the following way:

whereas the connection between h and t 2 has the form

Where With- speed of sound, which in calculations we will set equal to 340 m/sec.

Defining t 1 and t 2 of these expressions and substituting their values ​​into the formula connecting t 1 , t 2 and t, we obtain the irrational equation

From which you can find the height of the house.

In an approximate calculation (especially if the house is low), the second term on the left can be considered small and discarded. Then

Molecular physics

Task 3. Pencil

Exercise. Estimate the mechanical work that must be done in order to evenly raise a pencil floating in a vessel to the level of its lower end touching the surface of the water. Consider the position of the pencil to be vertical. Density of water With 0 = 1000 kg/m 3 .

Equipment: round pencil, almost full water bottle, ruler.

Possible solution. We lower the pencil into the bottle - it will float like a float, in accordance with Figure 5. Let L- the length of the entire pencil, V- its volume, h- the length of the part of the pencil immersed in water, V 1 - its volume, S- cross-sectional area and d- pencil diameter. Find the average density of a pencil With from the body floating condition:

With 0 gSh= сgSL, where With= With 0 hL.

Let's assume that we pull a pencil out of water at a constant speed using a dynamometer. When the pencil floats freely, the dynamometer shows zero. If the pencil is completely pulled out of the water, the dynamometer will show a force equal to the weight R pencil:

F = P = mg = сgV = с0hLgSL = с0hgрd24

It turns out that the readings of the dynamometer when pulling the pencil out of the water change from 0 to P according to the linear law, in accordance with Figure 6. In this case, the mechanical work A will be equal to the area of ​​the selected triangle:

A= 12Ph= With 0 h 2gрd 2 8.

For example, when h= 13,4 cm And d = 7,5 mm work is about 0.004 J.

Task 4. Alloy

Exercise. Determine the percentage (by weight) of tin in tin-lead solder. Assume that the volumes of lead and tin in the alloy are conserved. Lead Density With c = 11350 kg/m 3 , tin With 0 = 7300 kg/m 3 .

Equipment: ruler, weight (nut), cylindrical piece of solder, caliper or micrometer. Possible solution. This task is similar to Archimedes' task of determining the proportion of gold in the royal crown. However, for experiments, tin-lead solder is easier to obtain than corona.

Measuring the diameter of a piece of solder D and its length L, find the volume of a cylindrical piece of solder:

V =рD 2 L 4

We will determine the mass of solder by making lever scales. To do this, balance the ruler on the edge of the table (on a pencil, on a ballpoint pen, etc.). Then, using a nut of known mass, we balance a piece of solder on a ruler and, using the equality of the moments of force, we find the mass of the solder m. Let us write down the obvious equalities for the masses, volumes and densities of lead and tin:

m = m c +m o = ccV c +s o V o , V = V c +V o .

Solving these equations together, we find the volume of tin, its mass and its share in the total mass:

V o = rh o cV?mrh o c?rh oo , mo = с o V o , m o m = rh oo V o m

Problem 5. Surface tension

Exercise. Determine the coefficient of surface tension of water.

Equipment: plate, water, spoon, ruler, piece of straight aluminum wire 15-20 long cm and density 2700 kg/m 3 , micrometer, alcohol, cotton wool.

Possible solution. Pour an almost full plate of water. Place a wire on the edge of the plate so that one end touches the water and the other is outside the plate. The wire serves two functions: it is a lever scale and an analogue of the wire frame that is usually pulled out of the water to measure surface tension. Depending on the water level, different positions of the wire may be observed. The most convenient for calculations and measurements is the horizontal arrangement of the wire at a water level of 1-1.5 mm below the edge of the plate, in accordance with Figure 7. Using a spoon, you can adjust the level by adding or draining water. The wire should be pulled out of the plate until the film of water under the wire begins to break. In this extreme position the film has a height of 1.5-2 mm, and we can say that the surface tension forces applied to the wire are directed almost vertically downward.

Let m- mass of wire, L = L 1 + L 2 - wire length, m/L- mass per unit length of wire. Let us write down the condition for the equilibrium of the wire relative to the edge of the plate, i.e. equality of moments of forces:

F p (L 1 ?x 2)+m 1 gL 12 = m 2 gL 22 .

Let's substitute the surface tension force here F p =2x at, masses

m 1 =L 1 mL, m 2 = L 2 mL, m= cV= srd 2 L 4

and express the surface tension coefficient at. Measurements and calculations will be simplified if water wets the entire length L 1 . Finally we get

at= srd 2 g 8((LL 1 ?1) 2 ?1).

Quantities L And L 1 are measured with a ruler, and the diameter of the wire d- micrometer.

For example, when L = 15 cm, L 1 = 5,4 cm, d = 1,77 mm we get O = 0,0703 N/m, which is close to the table value of 0.0728 N/m.

Problem 6. Air humidity

Exercise. Determine the relative humidity in the room.

Equipment: glass room thermometer, household refrigerator, table of saturated water vapor pressures at different temperatures.

Possible solution. In the conventional method of measuring humidity, the object is cooled below its dew point and it "fogs up." Let's do the opposite. Refrigerator temperature (about +5 ° C) is much lower than the dew point for room air. Therefore, if you take a cooled glass thermometer out of the refrigerator, it will immediately “fog up” - the glass case will become opaque from moisture. Then the thermometer will begin to heat up, and at some point the condensed moisture on it will evaporate - the glass will become transparent. This is the dew point temperature, from which the relative humidity can be calculated using a table.

Problem 7. Evaporation

Exercise. Pour an almost full glass of water and place it in a warm place in the room so that the water evaporates faster. Measure the initial water level with a ruler and record the start time of the experiment. After a few days, the water level will drop due to evaporation. Measure the new water level and record the end time of the experiment. Determine the mass of water evaporated. On average, how many molecules escape from the surface of the water in 1 second? Approximately how many molecules are there on the surface of the water in the glass? Compare these two numbers. Take the diameter of a water molecule to be equal to d 0 = 0,3 nm. Knowing the specific heat of vaporization, determine the rate of heat transfer ( J/s) water from environment.

Possible solution. Let d- inner diameter of the glass, With- density of water, M - molar mass water, r- specific heat of vaporization, D h- decrease in water level over time t. Then the mass of evaporated water is

m= cv= With D hS= With D hрd 2 4.

This mass contains N = mN A /M molecules, where N A- Avogadro's constant. The number of molecules evaporated in 1 second is

N 1 = Nt= mN A Mt.

If S= pd 2/4 is the surface area of ​​water in a glass, and S 0 = pd 2 0 /4 is the cross-sectional area of ​​one molecule, then on the surface of water in a glass there is approximately

N 2 = SS 0 = (dd 0) 2 .

Water receives heat per unit time for evaporation

Qt= rmt.

If you make any calculations related to molecules, you always get interesting results. For example, let in time t= 5 days in a glass with diameter d = 65 mm the water level dropped by D h = 1 cm. Then we get that 33 turned into steam G water, for 1 With evaporated N 1 = 2.56?10 18 molecules, on the surface of the water in the glass there were N 2 = 4.69?1016 molecules, and 0.19 came from the environment W heat. The interesting thing is the attitude N 1 /N 2? 54, from which it is clear that for 1 With As many molecules evaporated as could fit in a glass in 54 layers of water.

Problem 8. Dissolution

Exercise. By pouring salt or sugar into boiling water, you will notice that the boiling stops for a short time due to the decrease in water temperature. Determine the amount of heat required to dissolve 1 kg baking soda in water at room temperature.

Equipment: homemade calorimeter, thermometer, water, soda, graduated cylinder (glass), load of known mass (nut weighing 10 G), plastic spoon.

Possible solution. The task includes an additional design task for the manufacture of a simple homemade calorimeter. For the internal vessel of the calorimeter, take a regular aluminum can with a volume of 0.33 liters. The top lid of the jar is removed so that an aluminum glass is obtained (weighing only 12 G) with a rigid upper rim. A slot is made inside the top rim so that the water can completely drain out of the jar. The outer plastic shell is made on the basis of a plastic bottle with a volume of 1.5 l. The bottle is cut into three parts, the upper part is removed, and the middle and lower parts are inserted into each other with some force and tightly fix the inner aluminum can in a vertical position. (If there is no calorimeter, then experiments can be carried out in a disposable plastic cup, the mass and heat transfer of which can be neglected).

First you need to make two measurements: 1) determine how much soda fits in a spoon (to do this, you need to look in a culinary reference book or “scoop out” a packet of soda of a known mass with this spoon); 2) decide on the amount of water - in a small amount of water the solution will immediately become saturated and part of the soda will not dissolve; in a large amount of water the temperature will change by fractions of a degree, which will complicate measurements.

Obviously, the amount of heat required to dissolve a substance is proportional to the mass of this substance: Q~m. To record equality, you should enter a proportionality coefficient, for example z, which can be called “specific heat of solution”. Then

Q= zm.

The dissolution of soda is carried out due to the energy released when the vessel with water cools. The value of z is found from the following heat balance equation:

mvcv(t 2 -t 1 )+ma cc (t 2 -t 1 ) = zm.

Where m v is the mass of water in the calorimeter, m a is the mass of the internal aluminum cup of the calorimeter, m- mass of dissolved soda, ( t 2 -t 1) - decrease in temperature in the calorimeter. The mass of the internal vessel of the calorimeter can be easily found using the rule of moments of forces, balancing the vessel and a load of known mass using a ruler and thread.

Measurements and calculations show that when m= 6 g and m v = 100 G the water cools down by 2-2.5 degrees C, and the value z turns out to be equal to 144-180 kJ/kg.

Task 9. Pot capacity

Exercise. How can you find the capacity of a pan using scales and a set of weights?

Clue. Weigh the empty pan, and then the pan with water.

Solution. Let the mass of the empty pan be m 1, and after filling with water it is m 2. Then the difference m 2 -m 1 gives the mass of water in the volume of the pan. Dividing this difference by the density of water With, find the volume of the pan:

Problem 10. How to separate the contents of a glass

Exercise. There is a cylindrical glass filled to the brim with liquid. How to divide the contents of a glass into two completely equal parts, having another vessel, but of a different shape and slightly smaller size?

Clue. Think about how you can draw a plane dividing the cylinder into two parts of equal volume.

Solution. If through points M And N mentally draw the plane as shown in Figure 1 A, then it will cut the cylinder into two symmetrical and therefore equal in volume figures, in accordance with Figure 8. From here follows the solution to the problem.

Gradually tilting the glass, you need to pour out the liquid it contains until the bottom just appears (Figure 1 b). At this moment, exactly half of the liquid will remain in the glass.

Electricity

Problem 11. Electric black box

A black box is an opaque, closed box that cannot be opened to examine its internal structure. Inside the box there are several electrical elements connected to each other in a simple electrical circuit. Typically, such elements are: current sources, fixed and variable resistors, capacitors, inductors, semiconductor diodes. There are several terminals on the outside of the box.

The main goal of the “black box” task: having made a minimum number of electrical measurements using external leads, “decipher” the “black box”, i.e.:

  • - establish which electrical devices are inside the “black box”.
  • - establish a diagram of their connection.
  • - determine the values ​​(values ​​of resistors, capacitances, etc.)

Exercise. Three resistors are connected to each other and placed in a “black box” with three terminals, in accordance with Figure 9. Exactly the same resistors are connected to each other in a different way and placed in a second “black box” with three terminals. Determine the resistance of each resistor. Jumpers are prohibited.

Equipment: multimeter.

Measuring the resistance between the terminals gave the following results:

Box No. 1: R 1-2 = 12Ohm, R 2-3 = 25Ohm, R 1-3 = 37Ohm

Box No. 2: R 1-2 = 5,45Ohm, R 2-3 = 15Ohm, R 1-3 = 20,45Ohm

Possible solution. There are four possible ways to connect three resistors to three external terminals so that the three measurements give different resistance values:

1) sequential, 2) mixed, 3) star, 4) triangle, in accordance with Figure 10.


We will show the sequence of searching for answers.

A characteristic feature of the first two schemes is that one of the measurements is equal to the sum of the other two, which corresponds to the conditions of the problem:

Consequently, in one box there is a serial connection, but then in the other there is a mixed connection, since the measurement results do not match, although the resistor values ​​are the same.

It is known that the relation is always satisfied

And since R 1-3 on the left more than R 1-3 on the right, then in the left box (No. 1) there is a serial connection, and in the right (No. 2) there is a mixed connection.

The series connection in the left box contains resistors with values ​​of 12 or 25 Ohm. Since neither one nor the other value is observed as part of a mixed connection, therefore, the value of one of the resistors R 1 = 15Ohm.

Other denominations: R 2 = 12Ohm And R 3 = 10Ohm.

Obviously, the same results can be reached using a different chain of reasoning.

Note also that 5 more combinations of schemes with two “black boxes” from the above four are possible. The most cumbersome mathematical part of the problem is to “decipher” the black box, which is known to contain a triangle.

In conclusion, we note that not everything can go as smoothly as in this example. Values ​​of resistance or other electrical quantities naturally contain errors. And, for example, the ratio can only be satisfied approximately.

Problem 12. Room temperature

Exercise. There is snow outside the window, but the room is warm. Unfortunately, there is nothing to measure the temperature with - there is no thermometer. But there is a battery, a very accurate voltmeter and the same ammeter, as much copper wire as you like and a detailed physical reference book. Is it possible to use them to find the air temperature in the room?

Clue. When a metal is heated, its resistance increases linearly.

Solution. Let's connect a battery in series, a coil of wire and turn on the ammeter so that it shows the voltage on the coil, in accordance with Figure 11. We record the instrument readings and calculate the resistance of the coil at room temperature:

After this, we will bring snow from the street, immerse a coil of it in it and, after waiting a little so that the snow begins to melt and the wire begins to warm up, we will determine the resistance of the wire in the same way R 0 at the temperature of melting snow, i.e. at 0 є WITH. Using then the relationship between the resistance of the conductor and its temperature

find the air temperature in the room:

The calculation uses the value temperature coefficient resistance b, taken from the reference book. In the room temperature range for pure copper b= 0,0043 hail - 1 . If the content of impurities in the copper from which the wire is made is not particularly high, and electrical measuring instruments have an accuracy class of 0.1, then the air temperature can be determined with an error of significantly less than one degree.

Optics

Problem 13.

Exercise. You need to find the radius of a spherical mirror (or radius of curvature concave lenses) using a stopwatch and a steel ball of known radius. How to do it?

Clue. The center of a ball rolling on the surface of a mirror makes the same motion as a pendulum.

Solution. Place the mirror horizontally and lower the ball onto it. If the ball is not lowered to the lowest point, it will begin to move along the surface of the mirror. It is not difficult to guess that if the ball moves without rotation (i.e. slides along the surface of the mirror), then its movement is completely similar to the movement of a pendulum with a suspension length R-r. Then from the pendulum formula

we can find the quantity we are interested in:

Period T determined using a stopwatch, and r known by condition.

Since the friction is usually high enough to cause the ball to move along the surface of the mirror with rotation, this solution does not agree well with experiment. In fact

Here is an example of a research task for the entire lesson.

Problem 14. Features of oscillation of a torsion pendulum.

Exercise. Explore the features of oscillation of a torsional pendulum and describe the main laws of its movement.

Equipment: tripod with coupling and foot, pieces of copper, steel and nichrome wire about 1m and various diameters, for example 0.3, 0.50, 0.65, 1.0 mm, thin light wooden stick 15-20 long cm, plasticine, paperclip, ruler, protractor, stopwatch.

The general appearance of the torsion pendulum should be in accordance with Figure 12. A paper clip, bent in a certain way, serves to balance the rod with weights. The pendulum, removed from the state of equilibrium, begins to perform a rotational-oscillatory motion.

In advance, you need to make pairs of balls of different weights from plasticine. The masses of the balls are proportional to the cube of their diameters, so it is possible to build a series, for example: m 1 = 1, m 2 = 2,5, m 3 = 5,2, m 3 = 6,8, m 4 = 8,3 rel. units

The diameter of the wires can be given to students in advance or they can be given the opportunity to make these measurements themselves using a caliper or micrometer.

Note. The success of the study largely depends on the correct selection of equipment, especially the diameters of the wires produced. In addition, it is desirable that the suspension of the torsion pendulum be in a tense state during the experiments, for which the masses of the loads must be large enough.

The subject of the study of a torsional pendulum follows from the assumption of the harmonic nature of its oscillations. The general list of experimental observations that can be carried out on this problem and on the proposed equipment is quite large. We present the simplest and most accessible ones.

  • - Does the period of oscillation depend on the amplitude (angle of rotation)?
  • - Does the period of oscillation depend on the length of the pendulum's suspension?
  • - Does the period of oscillation of a pendulum depend on the mass of the loads?
  • - Does the period of oscillation of a pendulum depend on the position of the weights on the rod?
  • - Does the period of oscillation depend on the diameter of the wire?

Naturally, it is required not only to answer the questions posed in monosyllables, but also to examine the nature of the expected dependencies.

Using the method of analogies, we put forward hypotheses about the oscillations of a torsion pendulum, comparing it with a mathematical pendulum studied by school curriculum. We take as a basis the period of oscillation and its dependence on various parameters of the pendulum. We outline the following hypotheses. Period of oscillation of a torsion pendulum:

At small angles of rotation it does not depend on the amplitude;

  • - proportional to the square root of the length of the suspension - T;
  • - proportional to the square root of the mass of the load - T;
  • - proportional to distance from the suspension center to the load centers - Tr;
  • - inversely proportional to the square of the wire diameter - T1/d 2 .

In addition, the oscillation period depends on the suspension material: copper, steel, nichrome. There are also a number of hypotheses here, we suggest testing them yourself.

1. We study the dependence of the period of oscillation of the pendulum on the amplitude (angle of rotation). The measurement results are presented in Table 3:

Table 3

Dependence of the period of oscillation of a pendulum on amplitude

L= 60cm, m = 8,3g, r = 12cm, d = 0,5mm

Conclusion. Within limits of up to 180, the dependence of the period of oscillation of the torsion pendulum on the amplitude is not detected. The scatter of measurement results can be explained by errors in measuring the oscillation period and random reasons.

To “open” other dependencies, you need to change only one parameter, leaving all others unchanged. Mathematical processing of results is best done graphically.

2. We study the dependence of the period of oscillation of the pendulum on its length: T = f(l). At the same time, we do not change m, r, d. The measurement results are presented in Table 4:

Table 4

Dependence of the period of oscillation of a pendulum on length

m = 8,3rel. units, r = 12cm, d = 0,5mm

Dependency graph T from l represents a curve of an increasing line, similar to a dependence, in accordance with Figure 13 A T 2 = l, in accordance with Figure 13, b.

Conclusion. The period of oscillation of a torsion pendulum is directly proportional to the square root of the length of the suspension. Some scatter of points can be explained by measurement errors in the period of oscillation and the length of the pendulum

3. We study the dependence of the period of oscillation of the pendulum on the mass of the loads: T=f(m). At the same time, we do not change l, r, d. The measurement results are presented in Table 5:

Table 5

Dependence of the period of oscillation of a pendulum on the mass of loads

l = 0,6m, r = 12cm, d = 0,5mm

Dependency graph T from m represents a curve of an increasing line, similar to a dependence, in accordance with Figure 14 A. To make sure of this, we build a dependency T 2 =f(m), according to Figure 14 b.

Conclusion. The period of oscillation of a torsion pendulum is directly proportional to the square root of the mass of the loads. Some scatter of points can be explained by measurement errors of the oscillation period and masses of the loads, as well as random reasons.

4. We study the dependence of the period of oscillation of the pendulum on the position of the weights: T = f(r). At the same time, we do not change l, m, d. The measurement results are presented in Table 6:

Table 6

Dependence of the period of oscillation of the pendulum on the position of the weights

m = 8,3rel. units, l = 0,6m, d = 0,5mm

Conclusion. The period of oscillation of a torsion pendulum is directly proportional to the distance r. Some scatter of points can be explained by measurement errors of the oscillation period and distance r, as well as random reasons.

We study the dependence of the period of oscillation of the pendulum on the diameter of the wire: T = f(d), in accordance with Figure 15 . At the same time we do not change m, r, l.

The measurement results are presented in Table 7.

Table 7

Dependence of the period of oscillation of a pendulum on the diameter of the wire

m = 8.3 relative units, r = 12 cm, l = 0.6 m

Dependency graph T from d represents a descending curve, in accordance with Figure 16 A. It can be assumed that this is a dependency where n= 1, 2, 3, etc. To check these assumptions, it is necessary to build graphs, etc. Of all such graphs, the most linear is the graph, in accordance with Figure 16 b.

Conclusion. The period of oscillation of a torsion pendulum is inversely proportional to the square of the diameter of the suspension wire. Some scatter of points can be explained by measurement errors of the oscillation period and wire diameter d, as well as random reasons.

The studies carried out allow us to conclude that the period of oscillation of a torsion pendulum should be calculated according to the formula, where k- proportionality coefficient, which also depends on the elastic properties of the suspension material - torsion modulus, shear modulus.

Home experimental tasks

Exercise 1.

Take a long, heavy book, tie it with a thin thread and

attach a 20 cm long rubber thread to the thread.

Place the book on the table and very slowly begin to pull on the end

rubber thread. Try to measure the length of the stretched rubber thread in

the moment the book begins to slide.

Measure the length of the stretched thread while moving the book evenly.

Place two thin cylindrical pens (or two

cylindrical pencil) and also pull the end of the thread. Measure the length

stretched thread with uniform movement of the book on the rollers.

Compare the three results obtained and draw conclusions.

Note. The next task is a variation of the previous one. It

also aimed at comparing static friction, sliding friction and friction

Task 2.

Place a hexagonal pencil on the book parallel to its spine.

Slowly lift the top edge of the book until the pencil begins to

slide down. Slightly reduce the tilt of the book and secure it in this way.

position by placing something under it. Now the pencil if its again

put it on a book, it won't move. It is held in place by frictional force -

static friction force. But it’s worth weakening this force a little - and for this it’s enough

click on the book with your finger, and the pencil will crawl down until it falls on

table. (The same experiment can be done, for example, with a pencil case, a match

box, eraser, etc.)

Think about why it is easier to pull a nail out of a board if you rotate it

around the axis?

To move a thick book on the table with one finger, you need to apply

some effort. And if you put two round pencils under the book or

handles, which in this case will be roller bearings, the book is easy

will move from a weak push with the little finger.

Carry out experiments and compare the static friction force and the friction force

sliding and rolling friction forces.

Task 3.

In this experiment, two phenomena can be observed at once: inertia, experiments with

Take two eggs: one raw and the other hard-boiled. Twist

both eggs on a large plate. You see that a boiled egg behaves differently,

than raw: it rotates much faster.

In a boiled egg, the white and yolk are tightly bound to their shell and

among themselves because are in a solid state. And when we spin

a raw egg, then we first unwind only the shell, only then, due to

friction, layer by layer rotation is transferred to the white and yolk. Thus,

liquid white and yolk, by their friction between the layers, slow down the rotation

shells.

Note. Instead of raw and boiled eggs, you can twist two pans,

one of which contains water, and the other contains the same volume of cereal.

Center of gravity. Exercise 1.

Take two faceted pencils and hold them parallel in front of you,

placing a ruler on them. Start bringing the pencils closer together. There will be rapprochement

occur in alternating movements: first one pencil moves, then the other.

Even if you want to interfere with their movement, you will not succeed.

They will still move in turns.

As soon as the pressure on one pencil became greater and the friction became so

the second pencil can now move under the ruler. But after a while

time the pressure above it becomes greater than above the first pencil, and because

As friction increases, it stops. And now the first one can move

pencil. So, moving one by one, the pencils will meet in the very middle

ruler at its center of gravity. This can be easily seen from the divisions of the ruler.

This experiment can also be done with a stick, holding it on outstretched fingers.

By moving your fingers, you will notice that they, also moving alternately, will meet

under the very middle of the stick. True, this is only special case. Try it

do the same with a regular floor brush, shovel or rake. You

you will see that your fingers do not meet in the middle of the stick. Try to explain

why is this happening.

Task 2.

This is an old, very visual experience. Do you have a pocket knife (folding)

probably a pencil too. Sharpen your pencil so it has a sharp end

and stick a half-opened pocket knife a little above the end. Put

pencil point on forefinger. Find such a position

half-open knife on a pencil, in which the pencil will stand on

finger, swaying slightly.

Now the question is: where is the center of gravity of a pencil and a pen

Task 3.

Determine the position of the center of gravity of a match with and without a head.

Place a matchbox on the table on its long narrow edge and

Place a match without a head on the box. This match will serve as a support for

another match. Take a match with its head and balance it on the support so that

so that it lies horizontally. Use a pen to mark the position of the center of gravity

matches with a head.

Scrape the head off the match and place the match on the support so that

The ink dot you marked was lying on the support. This is not for you now

succeed: the match will not lie horizontally, since the center of gravity of the match

moved. Determine the position of the new center of gravity and note that

Which side did he move? Mark with a pen the center of gravity of the match without

Bring a match with two points to class.

Task 4.

Determine the position of the center of gravity of the flat figure.

Cut out a figure of arbitrary (any fancy) shape from cardboard

and pierce several holes in different random places (it’s better if

they will be located closer to the edges of the figure, this will increase accuracy). Drive in

into a vertical wall or stand a small nail without a head or a needle and

hang a figure on it through any hole. Please note: figure

should swing freely on the nail.

Take a plumb line consisting of a thin thread and a weight and throw it

thread through the nail so that it points in the vertical direction

suspended figure. Mark the vertical direction on the figure with a pencil

Remove the figure, hang it from any other hole and again

Using a plumb line and a pencil, mark the vertical direction of the thread on it.

The point of intersection of the vertical lines will indicate the position of the center of gravity

of this figure.

Pass a thread through the center of gravity you have found, at the end of which

make a knot and hang the figure on this thread. The figure must hold

almost horizontal. The more accurately the experiment is done, the more horizontal it will be

hold on to the figure.

Task 5.

Determine the center of gravity of the hoop.

Take a small hoop (such as a hoop) or make a ring out of

flexible twig, made of a narrow strip of plywood or rigid cardboard. Hang

it onto the nail and lower the plumb line from the hanging point. When the thread plumb

calms down, mark on the hoop the points of her touching the hoop and between

use these points to tighten and secure a piece of thin wire or fishing line

(you need to pull it hard enough, but not so much that the hoop changes its

Hang the hoop on a nail at any other point and do the same

most. The point of intersection of the wires or lines will be the center of gravity of the hoop.

Note: the center of gravity of the hoop lies outside the substance of the body.

Tie a thread to the intersection of wires or lines and hang it on

she has a hoop. The hoop will be in indifferent equilibrium since the center

the gravity of the hoop and the point of its support (suspension) coincide.

Task 6.

You know that the stability of the body depends on the position of the center of gravity and

on the size of the support area: the lower the center of gravity and the larger the support area,

the more stable the body is.

Keeping this in mind, take a block or an empty matchbox and, placing it

alternately on squared paper at the widest, medium and widest

circle the smaller edge each time with a pencil to get three different

support area. Calculate the dimensions of each area in square centimeters

and write them down on paper.

Measure and record the height of the center of gravity position of the box for everyone

three cases (the center of gravity of the matchbox lies at the intersection

diagonals). Conclude at what position of the boxes is the most

sustainable.

Task 7.

Sit on a chair. Place your feet vertically without putting them under

seat. Sit completely straight. Try to stand up without bending forward,

without stretching your arms forward or moving your legs under the seat. You have nothing

If it works, you won't be able to get up. Your center of gravity, which is somewhere

in the middle of your body, will not allow you to stand up.

What condition must be met in order to stand up? You have to lean forward

or tuck your feet under the seat. When we get up, we always do both.

In this case, the vertical line passing through your center of gravity should

be sure to go through at least one of the soles of your legs or between them.

Then the balance of your body will be quite stable, you can easily

you can get up.

Well, now try to stand up, holding dumbbells or an iron in your hands. Pull

hands forward. You may be able to stand up without bending over or bending your legs under

Inertia. Exercise 1.

Place it on the glass postcard, and put a coin on the card

or a checker so that the coin is above the glass. Hit the postcard

click. The card should fly out and the coin (checker) should fall into the glass.

Task 2.

Place a double sheet of notebook paper on the table. One half

sheet, place a stack of books no lower than 25cm high.

Slightly lifting the second half of the sheet above the table level with both

With your hands, quickly pull the sheet towards you. The sheet should be freed from under

books, and the books must remain in place.

Place the book on the sheet of paper again and pull it now very slowly. Books

will move with the sheet.

Task 3.

Take a hammer, tie a thin thread to it, but let it

withstood the weight of the hammer. If one thread doesn't hold up, take two

threads Slowly lift the hammer up by the thread. The hammer will hang on

thread. And if you want to raise it again, but not slowly, but quickly

jerk, the thread will break (make sure that the hammer does not break when falling

nothing underneath). The inertia of the hammer is so great that the thread does not

survived. The hammer did not have time to quickly follow your hand, it remained in place, and the thread broke.

Task 4.

Take a small ball made of wood, plastic or glass. Make out

thick paper groove, place the ball in it. Move quickly across the table

groove and then suddenly stop it. The ball will continue by inertia

movement and will roll, jumping out of the groove.

Check where the ball will roll if:

a) pull the chute very quickly and stop it abruptly;

b) pull the chute slowly and stop suddenly.

Task 5.

Cut the apple in half, but not all the way through, and leave it hanging

Now hit the blunt side of the knife with the apple hanging on top

something hard, such as a hammer. Apple continuing to move along

inertia, will be cut and split into two halves.

The same thing happens when chopping wood: if it fails

split a block of wood, they usually turn it over and hit it with the butt as hard as they can

ax on a solid support. Churbak, continuing to move by inertia,

is impaled deeper on the ax and splits in two.

1. Explanatory note.

Physics teaching in high school is based on the physics course in primary school, subject to differentiation. The content of education should facilitate the implementation of a multi-level approach. Lyceum No. 44 aims to optimally develop the creative abilities of students with a special interest in the field of physics; this level of teaching is carried out in classes with in-depth study of physics.

The objects of study in a physics course at a level accessible to students, along with fundamental physical concepts and laws, should be experiment as a method of cognition, a method for constructing models and a method for their theoretical analysis. Lyceum graduates must understand the essence of models of natural objects (processes) and hypotheses, how theoretical conclusions are made, how to experimentally test models, hypotheses and theoretical conclusions.

At the lyceum, the number of hours in physics in advanced classes does not correspond to the new status of the physics and mathematics lyceum: in 9 classes - 2 hours. In this regard, it is proposed that technology lessons in grade 9 (1 hour per week, divided into two groups) be replaced with practical experimental physics in addition to the main lessons on the clock grid.

The purpose of the course is to provide students with the opportunity to satisfy their individual interest in studying practical applications of physics in the process of cognitive and creative activity while conducting independent experiments and research.

The main objective of the course is to help students make an informed choice of a profile for further education.

The program consists of the following parts: a) errors; b) laboratory work; c) experimental work; d) experimental tasks; d) testing.

In elective classes, schoolchildren will get to know in practice those types of activities that are leading in many engineering and technical professions related to the practical application of physics. The experience of independently performing first simple physical experiments, then tasks of research and design type will allow you to either verify the correctness of the preliminary choice, or change your choice and try yourself in some other direction.

At the same time, theoretical classes are advisable only at the first stage when forming a group and determining the interests and abilities of students.

The main forms of classes should be practical work by students in the physics laboratory and performing simple experimental tasks at home.

On practical exercises When performing laboratory work, students will be able to acquire the skills of planning a physical experiment in accordance with the task, learn to choose a rational measurement method, perform an experiment and process its results. Completing practical and experimental tasks will allow you to apply the acquired skills in a non-standard environment and become competent in many practical issues.

All types of practical tasks are designed to be used standard equipment physics classroom and can be performed in the form of laboratory work or as experimental elective tasks.

The elective course is aimed at instilling in schoolchildren self-confidence and the ability to use a variety of devices and household appliances in Everyday life, as well as the development of interest in a careful consideration of familiar phenomena and objects. The desire to understand, to understand the essence of phenomena, the structure of things that serve a person throughout his life, will inevitably require additional knowledge, push him to self-education, force him to observe, think, read, and invent.

Methods for measuring physical quantities (2 hours).

Basic and derived physical quantities and their measurements. Units and standards of quantities. Absolute and relative errors of direct measurements. Measuring instruments, tools, measures. Instrumental and reference errors. Instrument accuracy classes. Limits of systematic errors and methods for their assessment. Random measurement errors and estimation of their limits.

Stages of planning and performing an experiment. Precautions when conducting the experiment. Taking into account the influence of measuring instruments on the process under study. Selection of measurement method and measuring instruments.

Methods for monitoring measurement results. Recording measurement results. Tables and graphs. Processing of measurement results. Discussion and presentation of the results obtained.

Laboratory work (16 hours).

  1. Calculation of measurement errors of physical quantities.
  2. Study of uniformly accelerated motion.
  3. Determination of the acceleration of a body during uniformly accelerated motion.
  4. Measuring body weight.
  5. Study of Newton's second law.
  6. Determination of spring stiffness.
  7. Determination of sliding friction coefficient.
  8. Study of the motion of a body thrown horizontally.
  9. Study of the motion of a body in a circle under the influence of several forces.
  10. Clarification of the conditions of equilibrium of bodies under the influence of several forces.
  11. Determination of the center of gravity of a flat plate.
  12. Study of the law of conservation of momentum.
  13. Measuring the efficiency of an inclined plane.
  14. Comparison of the work done with the change in body energy.
  15. Study of the law of conservation of energy.
  16. Measuring the acceleration of gravity using a pendulum.

Experimental work (4 hours).

  1. Calculation of average and instantaneous speed.
  2. Measuring speed at the bottom of an inclined plane.
  3. Calculation and measurement of the speed of a ball rolling down an inclined chute.
  4. Study of oscillations of a spring pendulum.

Experimental tasks (10 hours).

  1. Solving experimental problems for grade 7 (2 hours).
  2. Solving experimental problems for grade 8 (2 hours).
  3. Solving experimental problems of grade 9 (2 hours).
  4. Solving experimental problems using a computer (4 hours).

Tested task (1 hour).

General lesson (1 hour).

3.Certification of students.

The credit form for assessing student achievements best corresponds to the characteristics of elective classes. It is advisable to give credit for laboratory work performed based on the submitted written report, which briefly describes the experimental conditions. The measurement results are presented in a systematic manner and conclusions are drawn.

Based on the results of completing creative experimental tasks, in addition to written reports, it is useful to practice reports in a general group lesson with a demonstration of completed experiments and manufactured devices. To conduct general results of the entire group’s classes, it is possible to hold a competition of creative works. At this competition, students will not only be able to demonstrate the experimental installation in action, but also talk about its originality and capabilities. Here it is especially important to format your report with graphs, tables, and briefly and emotionally talk about the most important things. In this case, it becomes possible to see and evaluate your work and yourself against the background of other interesting works and equally passionate people.

The student’s final grade for the entire elective course can be assessed, for example, according to the following criteria: completion of at least half of the laboratory work; performing at least one experimental task of a research or design type; Active participation in preparing and conducting seminars, discussions, competitions.

The proposed criteria for assessing student achievement can only serve as a guide, but are not mandatory. Based on his experience, the teacher can set other criteria.

4. Literature:

  1. Demonstration experiment in physics in high school./Ed. A. A. Pokrov
    sky. Part 1. - M.: Education, 1978.
  2. Methods of teaching physics in grades 7-11 of secondary school./Edited by V.P.
    Orekhova and A.V. Usova. - M.: Education, 1999.
  3. Martynov I.M., Khozyainova E.N. Didactic material on physics. 9th grade. - M.:
    Enlightenment, 1995.
  4. V.A.Burov, A.I.Ivanov, V.I.Sviridov. Frontal experimental tasks on
    Physics. 9th grade. – M: Prosveshchenie. 1988.
  5. Rymkevich A.P., Rymkevich P.A. Collection of physics problems for grades 9 – 11. – M.: About
    glow, 2000.
  6. Stepanova G.N. Collection of problems in physics: For grades 9-11 of general education
    decisions. - M.: Education, 1998.
  7. Gorodetsky D.N., Penkov I.A. Test work in physics. – Minsk “Highest”
    school”, 1987
  8. V.A. Burov, S.F. Kabanov, V.I. Sviridov. “Front-line experimental tasks on
    physics.” – M: Enlightenment.1988
  9. Kikoin I.K., Kikoin A.K. Physics: Textbook for 10 grades - M.: Education, 2003

T THEMATIC PLANNING FOR PHYSICS IN 9TH CLASS

Elective course: “Practical and experimental physics”

(in-depth study - 34 hours)

Stage – third

Level – advanced

Lesson type Watch Lesson content D/z
1 Lecture 1h Safety precautions. Abstract
2 Lecture 1h Errors in measurements of physical quantities. Abstract
3 Laboratory work № 1 1h Calculation of measurement errors of physical quantities Finish calculations
4 1h tasks
5 Experimental work 1h Calculation of average and instantaneous speed Finish calculations
6 Laboratory work No. 2 1h Study of uniformly accelerated motion Finish calculations
7 Laboratory work No. 3. 1 hour Determination of the acceleration of a body during uniformly accelerated motion. Finish calculations
8 Experimental work 1 hour Measuring speed at the bottom of an inclined plane. Finish calculations
9 Laboratory work No. 4 1h Measuring body mass Finish calculations
10 Laboratory work No. 5 1h Studying Newton's Second Law Finish calculations
11 Laboratory work No. 6 1 hour Determination of spring stiffness. Finish calculations
12 Laboratory work No. 7 1 hour Determination of sliding friction coefficient. Finish calculations
13 Laboratory work No. 8 1 hour Study of the motion of a body thrown horizontally. Finish calculations
14 Laboratory work No. 9 1 hour Study of the motion of a body in a circle under the influence of several forces.” Finish calculations
15 Solving experimental problems 1h Solving experimental problems for 7th grade tasks
16 Laboratory work No. 10 1 hour Clarification of the conditions of equilibrium of bodies under the influence of several forces. Finish calculations
17 Laboratory work No. 11 1 hour Determination of the center of gravity of a flat plate. Finish calculations
18 Solving experimental problems 1h tasks
19 Solving experimental problems 1h Solving experimental problems for 8th grade tasks
20 Laboratory work No. 12 1h Study of the law of conservation of momentum Finish calculations
21 Laboratory work No. 13 1h Inclined Plane Efficiency Measurement Finish calculations
22 Laboratory work No. 14 1 hour Comparison of the work done with the change in body energy” Finish calculations
23 Laboratory work No. 15 1h Studying the Law of Conservation of Energy Finish calculations
24 Experimental work 1h Calculation and measurement of the speed of a ball rolling down an inclined chute Finish calculations
25 Solving experimental problems 1h Tasks
26 Solving experimental problems 1h Solving experimental problems for grade 9 tasks
27 Experimental work 1h Study of oscillations of a spring pendulum Finish calculations
28 Laboratory work No. 16 1h Measuring the acceleration of free fall using a pendulum Finish calculations
29 1h Solving experimental problems for grade 9 Finish calculations
30 Solving experimental problems using a computer 1h Solving experimental problems using a computer Finish calculations
31 Solving experimental problems using a computer 1h Solving experimental problems using a computer Finish calculations
32 Solving experimental problems using a computer 1h Solving experimental problems using a computer Finish calculations
33 Tested task 1h Test
34 General lesson 1h Summing up and tasks for next year

LITERATURE:

  1. Demonstration experiment in physics in high school./Ed. A. A. Pokrovsky. Part 1. - M.: Education, 1978.
  2. Methods of teaching physics in grades 7-11 of secondary school./Edited by V.P. Orekhova and A.V. Usova. - M.: Education, 1999.
  3. Enochovich A.S. Handbook of Physics. - M.: Education, 1978.
  4. Martynov I.M., Khozyainova E.N. Didactic material on physics. 9th grade. - M.: Education, 1995.
  5. Skrelin L.I. Didactic material in physics. 9th grade. – M.: Education, 1998.
  6. Reader on Physics /Ed. B.I. Spassky. – M.: Education, 1982.
  7. Rymkevich A.P., Rymkevich P.A. Collection of physics problems for grades 9 – 11. – M.: Education, 2000.
  8. Stepanova G.N. Collection of problems in physics: For grades 9-11 educational institutions. - M.: Education, 1998.
  9. Gorodetsky D.N., Penkov I.A. Test work in physics. – Minsk “Higher School”, 1987.

Annex 1

Lesson No. 1: “Measurement of physical quantities and assessment of measurement errors.”

Lesson objectives: 1. To introduce students to the mathematical processing of measurement results and to teach how to present experimental data;

2. Development of computing abilities, memory and attention.

During the classes

The results of any physical experiment must be analyzed. This means that in the laboratory it is necessary to learn not only to measure various physical quantities, but also to check and find connections between them, to compare experimental results with the conclusions of theory.

But what does it mean to measure a physical quantity? What to do if the desired quantity cannot be measured directly and its value is found by the value of other quantities?

Measurement refers to the comparison of a measured quantity with another quantity taken as a unit of measurement.

The measurement is divided into direct and indirect.

In direct measurements, the value being determined is compared with a unit of measurement directly or using a measuring device calibrated in the appropriate units.

In indirect measurements, the required quantity is determined (calculated) from the results of direct measurements of other quantities that are related to the measured quantity by a certain functional relationship.

When measuring any physical quantity, you usually have to perform three sequential operations:

  1. Selection, testing and installation of devices;
  2. Observation of instrument readings and readings;
  3. Calculation of the required value from measurement results, assessment of errors.

Errors in measurement results.

The true value of a physical quantity is usually impossible to determine absolutely accurately. Each measurement gives the value of the determined quantity x with some error?x. This means that the true value lies in the interval

x measurement - dx< х ист < х изм + dх, (1)

where xmeas is the value of x obtained during the measurement; ?х characterizes the accuracy of x measurement. The quantity x is called the absolute error with which x is determined.

All errors are divided into systematic, random and misses (errors). The causes of errors are very diverse. Understanding the possible causes of errors and reducing them to a minimum - this means conducting an experiment correctly. Clearly this is not an easy task.

Systematic is an error that remains constant or changes naturally with repeated measurements of the same quantity.

Such errors arise as a result of the design features of the measuring instruments, the inaccuracy of the research method, any omissions of the experimenter, as well as when inaccurate formulas and rounded constants are used for calculations.

A measuring instrument is a device that is used to compare the measured value with a unit of measurement.

Any device contains one or another systematic error, which cannot be eliminated, but the order of which can be taken into account.

Systematic errors either increase or decrease the measurement results, that is, these errors are characterized by a constant sign.

Random errors are errors whose occurrence cannot be prevented.

Therefore, they can have a certain influence on a single measurement, but with repeated measurements they obey statistical laws and their influence on the measurement results can be taken into account or significantly reduced.

Slips and gross errors are excessively large errors that clearly distort the measurement result.

This class of errors is most often caused by incorrect actions of the observer. Measurements containing misses and gross errors should be discarded.

Measurements can be taken in terms of their accuracy technical And laboratory methods.

In this case, they are satisfied with such accuracy that the error does not exceed a certain predetermined value, determined by the error of the measuring equipment used.

With laboratory measurement methods, it is necessary to more accurately indicate the value of the measured quantity than is allowed by its single measurement using a technical method.

Then several measurements are made and the arithmetic mean of the obtained values ​​is calculated, which is taken as the most reliable value of the measured value. Then the accuracy of the measurement result is assessed (taking into account random errors).

From the possibility of carrying out measurements using two methods, it follows that there are two methods for assessing the accuracy of measurements: technical and laboratory.

Instrument accuracy classes.

To characterize most measuring instruments, the concept of reduced error E p (accuracy class) is often used.

The reduced error is the ratio of the absolute error?x to the limiting value xpr of the measured value (that is, its greatest value that can be measured on an instrument scale).

The given error, being essentially a relative error, expressed as a percentage:

E p = /dx/ x pr /*100%

According to the given error, devices are divided into seven classes: 0.1; 0.2; 0.5; 1.0; 1.5; 2.5; 4.

Devices of accuracy class 0.1; 0.2; 0.5 is used for precise laboratory measurements and is called precision.

In technology, devices of classes 1, 0 are used; 1.5; 2.5 and 4 (technical). The accuracy class of the device is indicated on the device scale. If there is no such designation on the scale, but this device is extracurricular, then its reduced error is more than 4%. In cases where the accuracy class is not indicated on the device, the absolute error is taken equal to half the value of the smallest division.

So, when measuring with a ruler, the smallest division of which is 1 mm, an error of up to 0.5 mm is allowed. For instruments equipped with a vernier, the instrument error is taken to be the error determined by the vernier (for a caliper - 0.1 mm or 0.05 mm; for a micrometer - 0.01 mm).

Appendix 2

Laboratory work: “Measuring the efficiency of an inclined plane.”

Equipment: wooden board, wooden block, tripod, dynamometer, measuring ruler.

Task: Investigate the dependence of the efficiency of an inclined plane and the gain in force obtained with its help from the angle of inclination of the plane to the horizon.

The efficiency of any simple mechanism is equal to the ratio of useful work A floor to perfect work A owl and is expressed as a percentage:

n = A sex / A owl *100% (1).

In the absence of friction, the efficiency of a simple mechanism, including an inclined plane, is equal to unity. In this case, the perfect work A of the force F t applied to the body and directed upward along the inclined plane is equal to useful work And the floor.

A gender = A owl.

Having designated the path traversed by the body along the inclined plane with the letter S, the height of the rise? , we get F*S=hgm.

In this case, the gain in strength will be equal to: k = gm/F=l/h.

In real conditions, the effect of friction reduces the efficiency of the inclined plane and reduces the gain in force.

To determine the efficiency of an inclined plane for the gain in force obtained with its help, the following expression should be used:

n = hgm/ F t l*100% (2), k = gm/F t (3).

The purpose of the work is to measure the efficiency of an inclined plane and the gain in force at different angles? its inclination to the horizon and explain the result obtained.

The order of work.

1. Assemble the installation according to Fig.1. Measure the height? and length l of the inclined plane (Fig. 2).

2. Calculate the maximum possible gain in force obtained for a given plane inclination (a=30).

3. Place the block on an inclined plane. After attaching the dynamometer to it, pull it up evenly along the inclined plane. Measure the traction force Ft.

4. Measure the gravity force mg of the block using a dynamometer and find the experimental value of the gain in force obtained using an inclined plane: k = gm/F t.

5. Calculate the efficiency of an inclined plane at a given angle of inclination

n = hgm/ F t l*100%

6. Repeat the measurements at other angles of inclination of the plane: a 2 =45?, a 3 =60?.

7. Enter the results of measurements and calculations into the table:

a m, kg h, m l, m F, N To n,%
1 30
2 45
3 60

8. Additional task

Compare the resulting theoretical dependence of n(a) and k(a) with the experimental results.

Control questions.

  1. What is the purpose of using an inclined plane?
  2. How can you increase the efficiency of an inclined plane?
  3. How can you increase the strength gain obtained using an inclined plane?
  4. Does the efficiency of an inclined plane depend on the mass of the load?
  5. Explain qualitatively the dependence of the efficiency of an inclined plane and the gain in force obtained with its help on the angle of inclination of the plane.

Appendix 3

List of experimental tasks for 7th grade

  1. Measuring the dimensions of the bar.
  2. Measuring the volume of liquid using a beaker.
  3. Measuring liquid density.
  4. Measuring the density of a solid.

All work is carried out with calculation of errors and verification

dimensions.

  1. Measuring body weight using a lever.
  2. Calculation of the gain in the strength of the tools used (scissors, wire cutters, pliers)
  3. Observation of the dependence of the kinetic energy of a body on its speed and mass.
  4. Find out what the friction force depends on experimentally.

List of experimental tasks for 8th grade

  1. Observation of the effects of electric current (thermal, chemical, magnetic and, if possible, physiological).
  2. Calculation of the characteristics of a mixed connection of conductors.
  3. Definition resistivity conductor with error assessment.
  4. Observation of the phenomenon of electromagnetic induction.
  1. Observation of energy absorption during ice melting.
  2. Observation of energy release during crystallization of hyposulfite.
  3. Observation of energy absorption during evaporation of liquids.
  4. Observation of the dependence of the rate of evaporation of a liquid on the type of liquid, its free surface area, temperature and rate of vapor removal.
  5. Determination of air humidity in the office.

List of experimental works for grade 9

  1. 1.Measurement of the modules of angular and linear velocities of a body during uniform motion in a circle.
  2. 2.Measurement of the modulus of centripetal acceleration of a body during uniform motion in a circle.
  3. 3. Observation of the dependence of the modules of the tension forces of the threads on the angle between them at a constant resultant force.
  4. 4. Study of Newton's third law.
  1. Observation of changes in the modulus of weight of a body moving with acceleration.
  2. Clarification of the equilibrium conditions for a body having an axis of rotation when forces act on it.
  3. Study of the law of conservation of momentum during elastic collision of bodies.
  4. Measuring the efficiency of the moving unit.

Appendix 4

Experimental tasks

Measuring the dimensions of the bar

Instruments and materials (Fig. 2): 1) measuring ruler, 2) wooden block.

Work order:

  • Calculate the scale division value of the ruler.
  • Specify the limit of this scale.
  • Measure the length, width, height of the block with a ruler.
  • Write down the results of all measurements in a notebook.

Measuring the volume of liquid using a beaker

Devices and materials (Fig. 3):

  • measuring cylinder (beaker),
  • glass of water.

Work order

  1. Calculate the scale division value of the beaker.
  2. Draw a part of the beaker scale in your notebook and make a note explaining the procedure for calculating the scale division price.
  3. Specify the limit of this scale.
  4. Measure the volume of water in the glass using a beaker. " "
  5. Write down the measurement result in your notebook.
  6. Pour the water back into the glass.

Pour, for example, 20 ml of water into a beaker. After checking by the teacher, add more water to it, bringing the level to the mark, for example, 50 ml. How much water was added to the beaker?

Liquid density measurement

Instruments and materials (Fig. 14): 1) training scales, 2) weights, 3) measuring cylinder (beaker), 4) glass of water.

Work order

  1. Write down: the value of the scale division of the beaker; the upper limit of the beaker scale.
  2. Measure the mass of the glass of water using a scale.
  3. Pour water from the glass into a beaker and measure the mass of the empty glass.
  4. Calculate the mass of water in the beaker.
  5. Measure the volume of water in the beaker.
  6. Calculate the density of water.

Calculation of body mass by its density and volume

Instruments and materials (Fig. 15): 1) training scales, 2) weights, 3) a measuring cylinder (beaker) with water, 4) an irregularly shaped body on a thread, 5) a table of densities.

Work order(Fig. 15)

  1. Measure your body volume using a beaker.
  2. Calculate body weight.
  3. Check your body weight calculation using a scale.
  4. Write down the results of measurements and calculations in your notebook.

Calculating the volume of a body based on its density and mass

Instruments and materials (Fig. 15): 1) training scales, 2) weights, 3) a measuring cylinder (beaker) with water, 4) an irregularly shaped body on a thread, b) a table of densities.

Work order

  1. Write down the substance that makes up the irregularly shaped body.
  2. Find the density of this substance in the table.
  3. Measure your body weight using a scale.
  4. Calculate the volume of the body.
  5. Check the result of calculating the volume of a body using a beaker.
  6. Write down the results of measurements and calculations in your notebook.

Study of the dependence of sliding friction force on the type of rubbing surfaces

Instruments and materials (Fig. 23): 1) dynamometer, 2) tribometer 3) weights with two hooks - 2 pcs., 4) sheet of paper, 5) sheet of sandpaper.

Work order

1. Prepare a table in your notebook to record the measurement results:

2. Calculate the value of the dynamometer scale division.
3.Measure the sliding friction force of a block with two loads:

4. Write down the measurement results in the table.

5. Answer the questions:

  1. Does sliding friction force depend on:
    a) from the type of rubbing surfaces?
    b) from the roughness of rubbing surfaces?
  2. In what ways can you increase and decrease the force of sliding friction? (Fig. 24):
    1) dynamometer, 2) tribometer.

Study of the dependence of sliding friction force on pressure force and independence from the area of ​​rubbing surfaces

Instruments and materials: 1) dynamometer, 2) tribometer; 3) weights with two hooks - 2 pcs.

Work order

  1. Calculate the value of the dynamometer scale division.
  2. Place a block with a large edge on the tribometer ruler, and a load on it and measure the sliding friction force of the block along the ruler (Fig. 24, a).
  3. Place a second weight on the block and again measure the sliding friction force of the block along the ruler (Fig. 24, b).
  4. Place the block with the smaller edge on the ruler, place two weights on it again and again measure the sliding friction force of the block along the ruler (Fig. 24, V)
  5. 5. Answer the question: does the force of sliding friction depend on:
    a) on the force of pressure, and if it depends, how?
    b) on the area of ​​the rubbing surfaces at a constant pressure force?

Measuring body weight using a lever

Equipment and materials: 1) lever-ruler, 2) measuring ruler, 3) dynamometer, 4) weight with two hooks, 5) metal cylinder, 6) tripod.

Work order

  1. Hang the arm on the axle attached to the tripod coupling. Rotate the nuts on the ends of the lever to set it to a horizontal position.
  2. Hang a metal cylinder to the left side of the lever, and a load to the right, having previously measured its weight with a dynamometer. Experimentally achieve balance between the lever and the load.
  3. Measure the arms of the forces acting on the lever.
  4. Using the lever equilibrium rule, calculate the weight of the metal cylinder.
  5. Measure the weight of the metal cylinder using a dynamometer and compare the result with the calculated one.
  6. Write down the results of measurements and calculations in your notebook.
  7. Answer the questions: will the result of the experiment change if:
  • balance the lever with a different arm length of the forces acting on it?
  • hang the cylinder on the right side of the lever, and the balancing weight on the left?

Calculation of the gain in the strength of instruments in which leverage is applied

"Device and materials (Fig. 45): 1) scissors, 2) wire cutters, 3) pliers, 4) measuring ruler.

Work order

  1. Familiarize yourself with the design of the tool offered to you, which uses a lever: find the axis of rotation, the points of application of forces.
  2. Measure the force arms.
  3. Calculate approximately within what limits the value can vary
    The toy is valid when using this tool.
  4. Write down the results of measurements and calculations in your notebook.
  5. Answer the questions:
  • How should the material to be cut be positioned in the scissors to obtain biggest win in force?
  • How should you hold the pliers in your hand to get the greatest gain in strength?

Observation of the dependence of the kinetic energy of a body on its speed and mass

Equipment and materials (Fig. 50): I) balls of different masses - 2 pcs., 2) trough, 3) block, 4) measuring tape, 5) tripod. Rice. 50.

Work order

  1. Support the gutter in an inclined position using a tripod, as shown in Figure 50. Place a wooden block at the lower end of the gutter
  2. Place a ball of smaller mass in the middle of the gutter and, releasing it, watch how the ball, rolling off the gutter and hitting a wooden block, moves the latter a certain distance, doing work to overcome the friction force.
  3. Measure the distance the block has moved.
  4. Repeat the experiment, launching the ball from the upper end of the chute, and again measure the distance over which the block has moved.
  5. Launch a ball of larger mass from the middle of the chute and measure the movement of the block again.

Measuring the modules of angular and linear velocities of a body during uniform motion in a circle

Equipment and materials* 1) a ball with a diameter of 25 mm on a thread 200 mm long, 2) a measuring ruler 30-35 cm with millimeter divisions, 3) a clock with a second hand or a mechanical metronome (one per class).

Work order

  1. Lift the ball by the end of the thread above the ruler and set it in uniform motion around the circle so that when rotating, each time it passes through the zero and, for example, tenth division of the scale (Fig. 9). To obtain stable movement of the ball, place the elbow of the hand holding the thread on the table
  2. Measure the time, for example, 30 full revolutions of the ball.
  3. Knowing the time of movement, the number of revolutions and the radius of rotation, calculate the absolute values ​​of the angular and linear velocities of the ball relative to the table.
  4. Write down the results of measurements and calculations in your notebook.
  5. Answer the questions:

Measuring the modulus of centripetal acceleration of a body during uniform circular motion

The equipment and materials are the same as in task 11.

Work order

  1. Follow steps 1, 2 tasks 11.
  2. Knowing the time of movement, the number of revolutions and the radius of rotation, calculate the module of the centripetal acceleration of the ball.
  3. Write down the results of measurements and calculations in your notebook:
  4. Answer the questions:
  • How will the module of the centripetal acceleration of the ball change if the number of its revolutions per unit time is doubled?
  • How will the module of the centripetal acceleration of the ball change if the radius of its rotation is doubled?

Observation of the dependence of the moduli of tension forces on the threads on the angle between them at a constant resultant force

Equipment and materials: 1) a weight weighing 100 g with two hooks, 2) training dynamometers - 2 pcs., 3) a thread 200 mm long with loops at the ends.

Work order


  • What are the moduli of tension forces of the threads? Did they change during the experience?
  • What is the modulus of the resultant of the two thread tension forces? Did it change during the experience?
  • What can be said about the dependence of the moduli of tension forces on the threads on the angle between them at a constant resultant force?

Studying Newton's Third Law

Equipment and materials: I) training dynamometers - 2 pcs., 2) thread 200 mm long with loops at the ends.

Work order


  • With what modulus force does the left dynamometer act on the right one? In which direction is this force directed? Which dynamometer is it attached to?
  • With what modulus force does the right dynamometer act on the left one? In which direction is this force directed? Which dynamometer is it attached to?

3. Increase dynamometer interaction. Notice their new readings.

4. Connect the dynamometers with a thread and tighten it.

5. Answer the questions:

  • With what modulus force does the left dynamometer act on the thread?
  • With what absolute force does the right dynamometer act on the thread?
  • With what modulus force does the thread stretch?

6. Draw a general conclusion from the experiments performed.

Observation of changes in the modulus of weight of a body moving with acceleration

Equipment and materials: 1) training dynamometer, 2) a weight weighing 100 g with two hooks, 3) a thread 200 mm long with loops at the ends.

Work order

  • Did the speed of the load change as it moved up and down?
  • How did the modulus of the weight of the load change as it moved rapidly up and down?

4. Place the dynamometer on the edge of the table. Tilt the load to the side at a certain angle and release (Fig. 18). Observe the dynamometer readings as the load oscillates.

5. Answer the questions:

  • Does the speed of the load change as it oscillates?
  • Do the acceleration and weight of the load change as it oscillates?
  • How do the centroidal acceleration and the weight of the load change as it oscillates?
  • At which points of the trajectory are the centripetal acceleration and the absolute weight of the load the greatest, and at which are the least? Figure 18.

Clarification of the equilibrium conditions of a body having an axis of rotation under the action of forces on it

Equipment and materials: 1) a sheet of cardboard measuring 150X150 mm with two thread loops, 2) training dynamometers - 2 pcs., 3) a sheet of cardboard measuring 240X340 mm with a nail driven in, 4) a student's square, 5) a measuring ruler 30-35 cm with millimeter divisions, 6) pencil.

Work order

1. Place a piece of cardboard over the nail. Hook the dynamometers onto the hinges, pull them with forces of approximately 2 and 3 N and position the hinges at an angle of 100-120° to each other, as shown in Figure 27. Make sure that the cardboard sheet returns to its state when tilted to the side

Rice. 27. Measure the modules of the applied forces (neglect the gravity of the cardboard).

2. Answer the questions:

  • How much force is acting on the cardboard?
  • What is the modulus of the resultant forces applied to the cardboard?

3. On a sheet of cardboard, draw straight line segments along which the forces act, and using a square, construct the shoulders of these forces, as shown in Figure 28.

4. Measure the force arms.

5. Calculate the moments of the acting forces and their algebraic sum. Under what condition is a body with a fixed axis of rotation in a state of equilibrium? Rice. 28. Write the answer in your notebook.

Study of the law of conservation of momentum during elastic collision of bodies

Equipment and materials: 1) balls with a diameter of 25 mm - 2 pcs., 2) thread 500 mm long, 3) a tripod for frontal work.

Work order

  • What is the total momentum of the balls before interaction?
  • Did the balls acquire the same impulses after interaction?
  • What is the total momentum of the balls after interaction?

4. Release the retracted ball and notice the deflection of the balls after impact. Repeat the experiment 2-3 times. Deflect one of the balls 4-5 cm from the equilibrium position, and leave the second one alone.

5. Answer the questions in step 3.

6. Draw a conclusion from the experiments done

Measuring the efficiency of the moving unit

Equipment and materials: 1) block, 2) training dynamometer, 3) measuring tape with centimeter divisions, 4) weights weighing 100 g with two hooks - 3 pcs., 5) tripod for frontal work, 6) thread 50 cm long with loops at the ends.

Work order

  1. Assemble the installation with a movable block, as shown in Figure 42. Throw a thread through the block. Hook one end of the thread onto the tripod leg, the other onto the hook of the dynamometer. Hang three weights weighing 100 g each from the block holder.
  2. Take the dynamometer in your hand, position it vertically so that the block with weights hangs on the threads, and measure the modulus of the tension force of the thread.
  3. Raise the loads evenly to a certain height and measure the modules of movement of the loads and the dynamometer relative to the table.
  4. Calculate the useful and perfect work relative to the table.
  5. Calculate the efficiency of the moving unit.
  6. Answer the questions:
  • What gain in strength does the movable block give?
  • Is it possible to get a gain in work using a moving block?
  • How to increase the efficiency of a moving unit?

Appendix5

Requirements for the level of training of secondary school graduates.

1. Master the methods of scientific knowledge.

1.1. Assemble experimental setups using a description, drawing or diagram and conduct observations of the phenomena being studied.

1.2. Measure: temperature, mass, volume, force (elasticity, gravity, sliding friction), distance, time interval, current strength, voltage, density, period of oscillation of the pendulum, focal length of the collecting lens.

1.3. Present measurement results in the form of tables, graphs and identify empirical patterns:

  • changes in body coordinates over time;
  • elastic forces from spring elongation;
  • current in the resistor versus voltage;
  • mass of a substance versus its volume;
  • body temperature versus time during heat exchange.

1.4. Explain the results of observations and experiments:

  • the change of day and night in the reference system associated with the Earth and in the reference system associated with the Sun;
  • greater compressibility of gases;
  • low compressibility of liquids and solids;
  • processes of evaporation and melting of matter;
  • evaporation of liquids at any temperature and its cooling during evaporation.

1.5. Apply experimental results to predict the values ​​of quantities characterizing the course of physical phenomena:

  • the position of the body when it moves under the influence of force;
  • extension of the spring under the action of a suspended load;
  • current strength at a given voltage;
  • the temperature value of the cooling water at a given point in time.

2. Know the basic concepts and laws of physics.

2.1. Define physical quantities and formulate physical laws.

2.2. Describe:

  • physical phenomena and processes;
  • changes and transformations of energy in the analysis of: free fall of bodies, movement of bodies in the presence of friction, oscillations of thread and spring pendulums, heating of conductors by electric current, melting and evaporation of matter.

2.3. Calculate:

  • resultant force using Newton's second law;
  • momentum of a body, if the speed of the body and its mass are known;
  • the distance over which sound travels in a certain time at a given speed;
  • kinetic energy bodies at given mass and speed;
  • potential energy of interaction of a body with the Earth and the force of gravity for a given body mass;
  • the energy released in a conductor during the passage of electric current (at a given current and voltage);
  • energy absorbed (released) when heating (cooling) bodies;

2.4. Construct an image of a point in a plane mirror and a converging lens.

3. Perceive, process and present educational information in various forms (verbal, figurative, symbolic).

3.1. Call:

  • sources of electrostatic and magnetic fields, methods of their detection;
  • energy conversion in internal combustion engines, electric generators, electric heating devices.

3.2. Give examples:

  • the relativity of the speed and trajectory of the same body in different reference systems;
  • change in the speed of bodies under the influence of force;
  • deformation of bodies during interaction;
  • manifestation of the law of conservation of momentum in nature and technology;
  • oscillatory and wave movements in nature and technology;
  • environmental consequences of the operation of internal combustion engines, thermal, nuclear and hydroelectric power plants;
  • experiments confirming the main provisions of the molecular kinetic theory.

3.4. Highlight main idea in the read text.

3.5. Find answers to the questions posed in the text you read.

3.6. Take notes on the text you read.

3.7. Define:

  • intermediate values ​​of quantities according to tables of measurement results and constructed graphs;
  • the nature of thermal processes: heating, cooling, melting, boiling (according to graphs of changes in body temperature over time);
  • resistance of the metal conductor (according to the oscillation graph);
  • according to the graph of coordinates versus time: to the coordinates of the body at a given point in time; periods of time during which the body moved at a constant, increasing, decreasing speed; time intervals of force action.

3.8. Compare the resistance of metal conductors (more - less) using graphs of current versus voltage.

Editor's Choice
To use presentation previews, create a Google account and sign in:...

William Gilbert formulated a postulate approximately 400 years ago that can be considered the main postulate of the natural sciences. Despite...

Functions of management Slides: 9 Words: 245 Sounds: 0 Effects: 60 The essence of management. Key concepts. Management Manager Key...

Mechanical period Arithmometer - a calculating machine that performs all 4 arithmetic operations (1874, Odner) Analytical engine -...
To use presentation previews, create a Google account and sign in:...
Preview: To use presentation previews, create a Google account and...
To use presentation previews, create a Google account and sign in:...
In 1943, Karachais were illegally deported from their native places. Overnight they lost everything - their home, their native land and...
When talking about the Mari and Vyatka regions on our website, we often mentioned and. Its origin is mysterious; moreover, the Mari (themselves...